The selling price of an article is equal to the cost of the

This topic has expert replies
Moderator
Posts: 7187
Joined: Thu Sep 07, 2017 4:43 pm
Followed by:23 members

Timer

00:00

Your Answer

A

B

C

D

E

Global Stats

The selling price of an article is equal to the cost of the article plus the markup. The markup on a certain television set is what percent of the selling price?

(1) The markup on the television set is 25 percent of the cost.
(2) The selling price of the television set is $250.

OA A

Source: Official Guide

User avatar
GMAT Instructor
Posts: 1449
Joined: Sat Oct 09, 2010 2:16 pm
Thanked: 59 times
Followed by:33 members

Timer

00:00

Your Answer

A

B

C

D

E

Global Stats

BTGmoderatorDC wrote:The selling price of an article is equal to the cost of the article plus the markup. The markup on a certain television set is what percent of the selling price?

(1) The markup on the television set is 25 percent of the cost.
(2) The selling price of the television set is $250.
\[{\text{sell}} = {\text{cost}} + {\text{mark}}\,\,\,\,\left( * \right)\]
\[\left[ {{\text{mark}} = \frac{x}{{100}}\left( {{\text{sell}}} \right)\,\,\,\, \Rightarrow } \right]\,\,\,\,\,\,\,?\,\, = \,\,100 \cdot \frac{{{\text{mark}}}}{{{\text{sell}}}}\,\,\,\,\,\,\, \Leftrightarrow \,\,\,\,\,\,\,\boxed{?\,\, = \frac{{{\text{mark}}}}{{{\text{sell}}}}}\,\,\]
\[\left( 1 \right)\,\,\,\frac{1}{4} = \frac{{{\text{mark}}}}{{{\text{cost}}}}\,\,\mathop = \limits^{\left( * \right)} \,\,\frac{{{\text{mark}}}}{{{\text{sell}} - {\text{mark}}}}\,\,\,\,\,\, \Rightarrow \,\,\,\,\,\,\frac{{{\text{sell}} - {\text{mark}}}}{{{\text{mark}}}} = 4\,\,\,\,\,\, \Rightarrow \,\,\,\,\,\,\frac{{{\text{sell}}}}{{{\text{mark}}}} - 1 = 4\,\,\,\,\,\, \Rightarrow \,\,\,\,\,\,?\,\, = \,\,{\left( {\frac{{{\text{sell}}}}{{{\text{mark}}}}} \right)^{ - 1}}\,\,\, = \frac{1}{5}\]\[\left( 2 \right)\,\,\,{\text{sell}} = 250\,\,\,\left\{ \begin{gathered}
\,{\text{If}}\,{\text{cost}} = 200\,\,\,\mathop \Rightarrow \limits^{\left( * \right)} \,\,\,\,{\text{mark}} = 50\,\,\,\,\, \Rightarrow \,\,\,\,\,? = \frac{{50}}{{250}} = \frac{1}{5}\,\, \hfill \\
\,{\text{If}}\,{\text{cost}} = 150\,\,\,\mathop \Rightarrow \limits^{\left( * \right)} \,\,\,\,{\text{mark}} = 100\,\,\,\,\, \Rightarrow \,\,\,\,\,? = \frac{{100}}{{250}} \ne \frac{1}{5}\,\, \hfill \\
\end{gathered} \right.\]

This solution follows the notations and rationale taught in the GMATH method.

Regards,
fskilnik.
Fabio Skilnik :: GMATH method creator ( Math for the GMAT)
English-speakers :: https://www.gmath.net
Portuguese-speakers :: https://www.gmath.com.br

GMAT/MBA Expert

User avatar
GMAT Instructor
Posts: 16207
Joined: Mon Dec 08, 2008 6:26 pm
Location: Vancouver, BC
Thanked: 5254 times
Followed by:1268 members
GMAT Score:770

by Brent@GMATPrepNow » Thu Sep 06, 2018 5:11 am

Timer

00:00

Your Answer

A

B

C

D

E

Global Stats

BTGmoderatorDC wrote:The selling price of an article is equal to the cost of the article plus the markup. The markup on a certain television set is what percent of the selling price?

(1) The markup on the television set is 25 percent of the cost.
(2) The selling price of the television set is $250.
Let C = cost of article

Target question: The markup on a certain television set is what percent of the selling price?

Statement 1: The markup on the television set is 25 percent of the cost.
If C = cost of article, markup = 0.25C
Selling price = cost + markup
= C + 0.25C
= 1.25C

So, the markup = 0.25C and the selling price = 1.25C
0.25C/1.25C = 0.25/1.25 = 1/5 = 20%
So, the markup on the TV is 20% of the selling price
Since we can answer the target question with certainty, statement 1 is SUFFICIENT

Statement 2: The selling price of the television set is $250
We have no information about the markup. So, consider these two possible cases:
Case a: The cost is $200 and the markup is $50. Here, the markup is $50 and the selling price is $250. So, the markup on the TV is 20% of the selling price
Case b: The cost is $150 and the markup is $100. Here, the markup is $100 and the selling price is $250. So, the markup on the TV is 40% of the selling price
Since we cannot answer the target question with certainty, statement 2 is NOT SUFFICIENT

Answer: A

Cheers,
Brent
Brent Hanneson - Creator of GMATPrepNow.com
Image

Legendary Member
Posts: 2229
Joined: Sun Oct 29, 2017 2:04 pm
Followed by:6 members

by swerve » Thu Sep 06, 2018 11:08 am

Timer

00:00

Your Answer

A

B

C

D

E

Global Stats

Markup = M Selling Price = P Cost = C
P=C+M

(1) M=0.25C
C=4M
Sub back into the equation
P=4M+M
P=5M

We want to find M/P which in this case is M/P = 1/5 or 20% so (1) is sufficient.

(2) 250 = M + C
We cannot solve, so the answer is A.